Last visit was: 25 Apr 2024, 21:43 It is currently 25 Apr 2024, 21:43

Close
GMAT Club Daily Prep
Thank you for using the timer - this advanced tool can estimate your performance and suggest more practice questions. We have subscribed you to Daily Prep Questions via email.

Customized
for You

we will pick new questions that match your level based on your Timer History

Track
Your Progress

every week, we’ll send you an estimated GMAT score based on your performance

Practice
Pays

we will pick new questions that match your level based on your Timer History
Not interested in getting valuable practice questions and articles delivered to your email? No problem, unsubscribe here.
Close
Request Expert Reply
Confirm Cancel
SORT BY:
Date
User avatar
Intern
Intern
Joined: 17 Jan 2010
Posts: 17
Own Kudos [?]: 134 [54]
Given Kudos: 8
Send PM
Most Helpful Reply
GMAT Club Legend
GMAT Club Legend
Joined: 19 Dec 2014
Status:GMAT Assassin/Co-Founder
Affiliations: EMPOWERgmat
Posts: 21846
Own Kudos [?]: 11667 [12]
Given Kudos: 450
Location: United States (CA)
GMAT 1: 800 Q51 V49
GRE 1: Q170 V170
Send PM
General Discussion
User avatar
SVP
SVP
Joined: 12 Oct 2009
Status:<strong>Nothing comes easy: neither do I want.</strong>
Posts: 2279
Own Kudos [?]: 3594 [1]
Given Kudos: 235
Location: Malaysia
Concentration: Technology, Entrepreneurship
Schools: ISB '15 (M)
GMAT 1: 670 Q49 V31
GMAT 2: 710 Q50 V35
Send PM
User avatar
Intern
Intern
Joined: 17 Nov 2009
Posts: 21
Own Kudos [?]: 168 [3]
Given Kudos: 9
Concentration: General Business Administration
Schools:University of Toronto, Mcgill, Queens
Send PM
Re: Which of the following has the greatest value? [#permalink]
2
Kudos
rahulms wrote:
which of the following has the greatest value?

A) \(\frac{\sqrt{2}}{\sqrt{3}} + \frac{\sqrt{3}}{\sqrt{4}} + \frac{\sqrt{4}}{\sqrt{5}} + \frac{\sqrt{5}}{\sqrt{6}}\)

B) \(\frac{2}{3} + \frac{3}{4} + \frac{4}{5} + \frac{5}{6}\)

C) \(\frac{2^2}{3^2} + \frac{3^2}{4^2} + \frac{4^2}{5^2} + \frac{5^2}{6^2}\)

D) \(1 - \frac{1}{3} + \frac{4}{5} - \frac{3}{4}\)

E) \(1 - \frac{3}{4} + \frac{4}{5} + \frac{1}{3}\)



One of this type question is also in the GMATclub tests as well i didn't remember which one it was. However in order to solve the greatest value for the options available we have to find out the lowest denominator of the options available and use POE method.

A) \(\frac{\sqrt{2}}{\sqrt{3}} + \frac{\sqrt{3}}{\sqrt{4}} + \frac{\sqrt{4}}{\sqrt{5}} + \frac{\sqrt{5}}{\sqrt{6}}\)
B) \(\frac{2}{3} + \frac{3}{4} + \frac{4}{5} + \frac{5}{6}\)
C) \(\frac{2^2}{3^2} + \frac{3^2}{4^2} + \frac{4^2}{5^2} + \frac{5^2}{6^2}\)

For all these options it is quite evident that option A has the lowest denominator so it will result in greater value because of the root.

D) \(1 - \frac{1}{3} + \frac{4}{5} - \frac{3}{4}\)
E) \(1 - \frac{3}{4} + \frac{4}{5} + \frac{1}{3}\)
if you quickly solve D and E will reveal that denominator is greater than the option A

So it must be A
User avatar
Manager
Manager
Joined: 20 Apr 2010
Posts: 154
Own Kudos [?]: 248 [0]
Given Kudos: 28
Concentration: Finacee, General Management
Schools:ISB, HEC, Said
 Q48  V28
Send PM
Re: Which of the following has the greatest value? [#permalink]
It seems to be easy but time consuming. What is the source of this problem.
User avatar
Manager
Manager
Joined: 20 Apr 2010
Posts: 154
Own Kudos [?]: 248 [0]
Given Kudos: 28
Concentration: Finacee, General Management
Schools:ISB, HEC, Said
 Q48  V28
Send PM
Re: Which of the following has the greatest value? [#permalink]
This seems to be simple but time consuming problem what is the source?
User avatar
Manager
Manager
Joined: 25 Feb 2010
Posts: 208
Own Kudos [?]: 320 [1]
Given Kudos: 10
Send PM
Re: Which of the following has the greatest value? [#permalink]
prashantbacchewar wrote:
This seems to be simple but time consuming problem what is the source?


If its taking more time then try this:

Try to find out the option which has the lowest denominator so it will result in greater value.
User avatar
Retired Moderator
Joined: 02 Sep 2010
Posts: 615
Own Kudos [?]: 2931 [2]
Given Kudos: 25
Location: London
 Q51  V41
Send PM
Re: Which of the following has the greatest value? [#permalink]
2
Kudos
prashantbacchewar wrote:
This seems to be simple but time consuming problem what is the source?


I really think this is one of those problems which can throw you off because it seems complicated, but I do believe its one of those questions you can easily do in less than 30 secs ... and its this kind of stuff that will make you save time in the real GMAT

Between A,B,C --> A is clearly the biggest

D is clearly less than B, it has the same terms some positive and some negative

E is also less than B : 4/5 - 3/4 + 2/3 + 2/3 which is less than 2/3-3/4+4/5+5/6, which has to be less than B

No calculations, just a quick manipulation mentally is sufficient
User avatar
Manager
Manager
Joined: 20 Apr 2010
Posts: 154
Own Kudos [?]: 248 [0]
Given Kudos: 28
Concentration: Finacee, General Management
Schools:ISB, HEC, Said
 Q48  V28
Send PM
Re: Which of the following has the greatest value? [#permalink]
shrouded1

Thanks for sharing approach +1 to you
User avatar
Retired Moderator
Joined: 04 Oct 2009
Status:2000 posts! I don't know whether I should feel great or sad about it! LOL
Posts: 767
Own Kudos [?]: 3947 [0]
Given Kudos: 109
Location: Peru
Concentration: Finance, SMEs, Developing countries, Public sector and non profit organizations
Schools:Harvard, Stanford, Wharton, MIT &amp; HKS (Government)
GPA: 4.0
WE 1: Economic research
WE 2: Banking
WE 3: Government: Foreign Trade and SMEs
Send PM
Re: Which of the following has the greatest value? [#permalink]
+1 A

I think that here is not necessary to make many calculations.
You can eliminate some options with just watching.
VP
VP
Joined: 12 Dec 2016
Posts: 1030
Own Kudos [?]: 1779 [0]
Given Kudos: 2562
Location: United States
GMAT 1: 700 Q49 V33
GPA: 3.64
Send PM
Re: Which of the following has the greatest value? [#permalink]
E is less than B because E = 1/4 + 1/3 + 4/5 < 2/3 + 4/5 => E <B
Tutor
Joined: 16 Oct 2010
Posts: 14823
Own Kudos [?]: 64925 [3]
Given Kudos: 426
Location: Pune, India
Send PM
Re: Which of the following has the greatest value? [#permalink]
2
Kudos
1
Bookmarks
Expert Reply
rahulms wrote:
Which of the following has the greatest value?

A) \(\frac{\sqrt{2}}{\sqrt{3}} + \frac{\sqrt{3}}{\sqrt{4}} + \frac{\sqrt{4}}{\sqrt{5}} + \frac{\sqrt{5}}{\sqrt{6}}\)

B) \(\frac{2}{3} + \frac{3}{4} + \frac{4}{5} + \frac{5}{6}\)

C) \(\frac{2^2}{3^2} + \frac{3^2}{4^2} + \frac{4^2}{5^2} + \frac{5^2}{6^2}\)

D) \(1 - \frac{1}{3} + \frac{4}{5} - \frac{3}{4}\)

E) \(1 - \frac{3}{4} + \frac{4}{5} + \frac{1}{3}\)



All terms of each option are 1 or less than 1. Options (D) and (E) have negative terms and a quick look shows that they will certainly be less than the other 3 options. So ignore them.

Out of (B) and (C), compare first terms of each: 2/3 (= 6/9) of option (B) is more than 4/9 of option (C). All other terms will follow the same pattern so (B) is greater than (C)

Now compare the first terms of options (A) and (B): \(\sqrt{2}/\sqrt{3} ( = \sqrt{6}/3\) = 2.something / 3) while first term of option (B) is just 2/3. So the first term of option (A) is greater. All other terms will follow the same pattern.

Hence option (A) is greatest.
SVP
SVP
Joined: 24 Nov 2016
Posts: 1720
Own Kudos [?]: 1344 [0]
Given Kudos: 607
Location: United States
Send PM
Re: Which of the following has the greatest value? [#permalink]
rahulms wrote:
Which of the following has the greatest value?

A) \(\frac{\sqrt{2}}{\sqrt{3}} + \frac{\sqrt{3}}{\sqrt{4}} + \frac{\sqrt{4}}{\sqrt{5}} + \frac{\sqrt{5}}{\sqrt{6}}\)

B) \(\frac{2}{3} + \frac{3}{4} + \frac{4}{5} + \frac{5}{6}\)

C) \(\frac{2^2}{3^2} + \frac{3^2}{4^2} + \frac{4^2}{5^2} + \frac{5^2}{6^2}\)

D) \(1 - \frac{1}{3} + \frac{4}{5} - \frac{3}{4}\)

E) \(1 - \frac{3}{4} + \frac{4}{5} + \frac{1}{3}\)


D vs. E: E > D
\(-\frac{1}{3}+ \frac{4}{5}<\frac{4}{5} + \frac{1}{3}\)

B vs. E: B > E
\(\frac{2}{3} + \frac{3}{4} + \frac{4}{5}>\frac{4}{5} + \frac{1}{3} + \frac{1}{4}\)

B vs. C: B > C
A proper fraction raised to a positive number gets smaller.

B vs. A: A > B
When we take the root of a proper fraction it gets larger.

Ans (A)
User avatar
Non-Human User
Joined: 09 Sep 2013
Posts: 32681
Own Kudos [?]: 822 [0]
Given Kudos: 0
Send PM
Re: Which of the following has the greatest value? [#permalink]
Hello from the GMAT Club BumpBot!

Thanks to another GMAT Club member, I have just discovered this valuable topic, yet it had no discussion for over a year. I am now bumping it up - doing my job. I think you may find it valuable (esp those replies with Kudos).

Want to see all other topics I dig out? Follow me (click follow button on profile). You will receive a summary of all topics I bump in your profile area as well as via email.
GMAT Club Bot
Re: Which of the following has the greatest value? [#permalink]
Moderators:
Math Expert
92915 posts
Senior Moderator - Masters Forum
3137 posts

Powered by phpBB © phpBB Group | Emoji artwork provided by EmojiOne